سوال المپیاد ریاضی

مدیران انجمن: parse, javad123javad

ارسال پست
نمایه کاربر
rohamavation

نام: roham hesami radرهام حسامی راد

محل اقامت: 100 مایلی شمال لندن جاده آیلستون، لستر، لسترشر. LE2

عضویت : سه‌شنبه ۱۳۹۹/۸/۲۰ - ۰۸:۳۴


پست: 3268

سپاس: 5491

جنسیت:

تماس:

سوال المپیاد ریاضی

پست توسط rohamavation »

در نمودار زیر AD عمود بر AC و ∠BAD=∠DAE=12∘ است. اگر AB+AE=BC، ∠ABC را پیدا کنید. تصویر
من در یک مسابقه المپیاد ریاضی با این سوال برخورد کردم و مطمئن نیستم چگونه آن را حل کنم.
اثبات
F را روی $\overrightarrow{BA}$ فراتر از A بگیرید به طوری که BF=BC
AE=AF و △AEF متساوی الساقین است.
2∠BAD=∠BAE=∠AEF+∠AFE=2∠AFE
بنابراین AD موازی با EF است.
AD⊥AC و سپس AC⊥EF، که به ما می گویند □AECF بادبادک است.
∠EFC=90∘−∠FCA، ∠AFC=∠AFE+∠CFE=102∘−∠FCA
∠AFC=∠ECF=2∠FCA، زیرا △BCF متساوی الساقین است.
بنابراین، ∠FCA=34∘ و ∠ABC=44∘
تصویر

نمایه کاربر
rohamavation

نام: roham hesami radرهام حسامی راد

محل اقامت: 100 مایلی شمال لندن جاده آیلستون، لستر، لسترشر. LE2

عضویت : سه‌شنبه ۱۳۹۹/۸/۲۰ - ۰۸:۳۴


پست: 3268

سپاس: 5491

جنسیت:

تماس:

Re: سوال المپیاد ریاضی

پست توسط rohamavation »

اگر p، q و r اعداد اول باشند به طوری که حاصلضرب آنها 19 برابر مجموع آنها باشد، $p^2$ + $q^2$ + $r^2$ را پیدا کنید.
من در یک مسابقه المپیاد ریاضی با این سوال مواجه شدم و نمی دانستم چگونه آن را انجام دهم. کسی میتونه کمک کنه؟
یکی از اعداد اول باید 19 باشد، بنابراین WLOG r=19$. $سپس (p−1)(q−1)=20
اگر p، q و r اعداد اول باشند به طوری که حاصلضرب آنها 19 برابر مجموع آنها باشد، $p^2$ + $q^2$ + $r^2$ را پیدا کنید.
این عبارت را می توان اینگونه نوشت: pqr=19(p+q+r)
از آنجایی که p، q، r اعداد اول هستند، حاصلضرب آن فقط p، q و r به عنوان مقسوم‌کننده‌های اول خواهد بود. بنابراین، یکی از آنها باید 19 باشد. بگویید r=19. سپس، ما داریم:
pq19=19(p+q+19)
pq=p+q+19
pq−p−q=19 (*)
از طرف دیگر، ما همیشه عبارت را داریم: (p−1)(q−1)=pq−p−q+1
که معادل است با:
(p-1)(q-1)-1=pq-p-q
پیوستن با (*)(p-1)(q-1)-1=19
(p-1) (q-1) = 20
20 را می توان در موارد زیر فاکتور گرفت: 20=2∗2∗5
در بلوک های دوتایی:20=4*5
یا20=2∗10یا20=1∗20بنابراین ما باید گزینه هایی را انتخاب کنیم:
اولین
p−1=4−>p=5
q−1=5−>q=6
این نمی تواند اتفاق بیفتد، زیرا p، q، r باید اول باشند و p=6 نیست.
دومین
p−1=1−>p=2
q−1=20−>q=21
این نمی تواند اتفاق بیفتد، زیرا p، q، r باید اول باشند و q=21 نیست.
سوم
p−1=2−>p=3
q−1=10−>q=11
هر دو p=3 و q=11 اعداد اول هستند، بنابراین می توانیم جلو برویم. در چنین حالتی،
$p^2 + q^2 + r^2 = 3^2 + 11^2 + 19^2 = 9 + 121 + 361 = 491$
تصویر

نمایه کاربر
rohamavation

نام: roham hesami radرهام حسامی راد

محل اقامت: 100 مایلی شمال لندن جاده آیلستون، لستر، لسترشر. LE2

عضویت : سه‌شنبه ۱۳۹۹/۸/۲۰ - ۰۸:۳۴


پست: 3268

سپاس: 5491

جنسیت:

تماس:

Re: سوال المپیاد ریاضی

پست توسط rohamavation »

اگر مجموع معکوس های مثبت 4 عدد صحیح مثبت برابر با 1.1 باشد، کمترین مجموع اعداد صحیح چقدر است؟
با توجه به چهار عدد صحیح مثبت مختلف $A, B, C, D$ به طوری که$\frac{1}{A}+\frac{1}{B}+ \frac{1}{C}+\frac{1}{D}=1.1$ کوچکترین مقدار ممکن A+B+C+D را پیدا کنید.
آیا مقدار 1.1 به چیزی ربطی دارد؟ همچنین برای حل این سوال از چه ترفندی استفاده کنم؟ آیا فرمول معادله معکوس از نوعی وجود دارد که بتوانم از آن استفاده کنم؟
من سعی کردم این کار را انجام دهم:
$\frac{1}{A}+\frac{1}{B}+ \frac{1}{C}+\frac{1}{D}=1.1$
$ABC+BCD+ACD+ABD=1.1ABCD$
اما حالا نمی دانم کجا ادامه دهم.

بدون از دست دادن کلیت، فرض کنید$A < B < C < D$
توجه داشته باشید که اگر A≥3$،$ سپس B≥4، C≥5، D≥6$، $و سپس $\tfrac{1}{A}+\tfrac{1}{B}+\tfrac{1}{C}+\tfrac{1}{D} \le \tfrac{1}{3}+\tfrac{1}{4}+\tfrac{1}{5}+\tfrac{1}{6} = \tfrac{19}{20} < 1.1$. بنابراین ما به A=1 یا A=2 نیاز داریم.
مورد 1: A=2. سپس به $\tfrac{1}{B}+\tfrac{1}{C}+\tfrac{1}{D} = \tfrac{3}{5}$ نیاز داریم.
از آنجایی که $\tfrac{3}{B} > \tfrac{1}{B}+\tfrac{1}{C}+\tfrac{1}{D} = \tfrac{3}{5}$، باید B<5، یعنی B=3 یا B=4 داشته باشیم.
اگر B=4 به $\tfrac{1}{C}+\tfrac{1}{D} = \tfrac{7}{20}$ با $4 < C < D$ نیاز داریم. از آنجایی که $\tfrac{2}{C} > \tfrac{1}{C}+\tfrac{1}{D} = \tfrac{7}{20}$، باید$4 < C < D$، یعنی C≤5 داشته باشیم. از آنجایی که $4 < C \le 5$ باید C=5 داشته باشیم، اما بعد از آن $D = \tfrac{20}{3}$ که یک عدد صحیح نیست. بنابراین هیچ راه حلی با A=2 و B=4 وجود ندارد.
اگر B=3 به$\tfrac{1}{C}+\tfrac{1}{D} = \tfrac{4}{15}$ با 3<C<D نیاز داریم. از آنجایی که$\tfrac{2}{C} > \tfrac{1}{C}+\tfrac{1}{D} = \tfrac{4}{15}$، باید$C < \tfrac{15}{2}$، یعنی C≤7 داشته باشیم. آزمایش C=4،5،6،7 به ترتیب $D = 60, 15, 10, \tfrac{105}{13}$ را به دست می دهد. در این مورد، کوچکترین مجموع که C و D اعداد صحیح هستند 21 است که برای (A,B,C,D)=(2,3,6,10) رخ می دهد.
مورد 2: A=1. سپس به $\tfrac{1}{B}+\tfrac{1}{C}+\tfrac{1}{D} = \tfrac{1}{10}$ نیاز داریم. اما از آنجایی که $\tfrac{3}{D} < \tfrac{1}{B}+\tfrac{1}{C}+\tfrac{1}{D} = \tfrac{1}{10}$، هر راه حلی در این حالت D>30 خواهد داشت و بنابراین، $A+B+C+D > 30 > 21$ خواهد بود. .
بنابراین، حاقل جمع 21 است.
توجه داشته باشید که اگر فقط نیاز به دریافت سریع پاسخ بدون اثبات دقیق دارید، احتمالاً می توانید حدس بزنید و بررسی کنید تا زمانی که چیزی نسبتاً کوچک پیدا کنید. در مسائل مربوط به کسرهای مصری (کسری با عدد 1)، مجموع $\tfrac{1}{2}+\tfrac{1}{3}+\tfrac{1}{6} = 1$ بسیار به دست می آید، یعنی کوچکترین مجموعه ای از کسرهای مصری مجزا است که جمع آنها 1 می شود. بنابراین ساختن آن چندان سخت نیست. از آن به $\tfrac{1}{2}+\tfrac{1}{3}+\tfrac{1}{6}+\tfrac{1}{10} = \tfrac{11}{10}$می رسد. من مطمئن نیستم که آیا راه آسانی برای متقاعد کردن خود وجود دارد



من به سادگی تمام احتمالات را به زور اعمال می کنم. A<B<C<D را فرض کنید.
اگر A≥3، آنگاه $\frac{1}{A} + \frac{1}{B} + \frac{1}{C} + \frac{1}{D} \leq \frac{1}{3} + \frac{1}{4} + \frac{1}{5} + \frac{1}{6} < 0.34 + 0.25 + 0.2 + 0.17 = 0.96 < 1.1$
فرض کنید A=2.
اگر B≥5، $\frac{1}{A} + \frac{1}{B} + \frac{1}{C} + \frac{1}{D} \leq \frac{1}{2} + \frac{1}{5} + \frac{1}{6} + \frac{1}{7} < 0.5 + 0.2 + 0.17 + 0.15 = 1.02 < 1.1$.
فرض کنید B=4.
اگر $\frac{1}{A} + \frac{1}{B} + \frac{1}{C} + \frac{1}{D} \leq \frac{1}{2} + \frac{1}{4} + \frac{1}{6} + \frac{1}{7} < 0.5 + 0.25 + 0.17 + 0.15 = 1.07 < 1.1$
اگر C=5 باشد، $\frac{1}{D} = 1.1 - \frac{1}{A} - \frac{1}{B} - \frac{1}{C} = \frac{3}{20}$. در این حالت D یک عدد صحیح نیست.
فرض کنید B=3.
اگر $\frac{1}{A} + \frac{1}{B} + \frac{1}{C} + \frac{1}{D} \leq \frac{1}{2} + \frac{1}{3} + \frac{1}{8} + \frac{1}{9} < 0.5 + 0.34 + 0.13 + 0.12 = 1.09 < 1.1$
اگر $\frac{1}{D} = 1.1 - \frac{1}{A} - \frac{1}{B} - \frac{1}{C} = \frac{13}{105}$. در این حالت D یک عدد صحیح نیست.
اگر C=6 باشد، $\frac{1}{D} = 1.1 - \frac{1}{A} - \frac{1}{B} - \frac{1}{C} = \frac{1}{10}$. بنابراین (A,B,C,D)=(2,3,6,10) جواب معادله است.
اگر C=5 باشد، $\frac{1}{D} = 1.1 - \frac{1}{A} - \frac{1}{B} - \frac{1}{C} = \frac{1}{15}$ بنابراین (A,B,C,D)=(2,3,5,15) جواب معادله است.
اگر C=4 باشد، $\frac{1}{D} = 1.1 - \frac{1}{A} - \frac{1}{B} - \frac{1}{C} = \frac{1}{60}$. پس (A,B,C,D)=(2,3,4,60) جواب معادله است.
فرض کنید A=1. سپس
$\frac{1}{B} + \frac{1}{C} + \frac{1}{D} = \frac{1}{10}$
اگر D<30، پس
$\frac{1}{B} + \frac{1}{C} + \frac{1}{D} > \frac{1}{30} + \frac{1}{30} + \frac{1}{30} = \frac{1}{10}.$
بنابراین D باید حداقل 30 باشد و بنابراین $(A, B, C, D) = (2, 3, 6, 10)$
در نتیجه، حداقل جمع 21 است که با (A,B,C,D)=(2,3,6,10) به دست می آید.
تصویر

نمایه کاربر
rohamavation

نام: roham hesami radرهام حسامی راد

محل اقامت: 100 مایلی شمال لندن جاده آیلستون، لستر، لسترشر. LE2

عضویت : سه‌شنبه ۱۳۹۹/۸/۲۰ - ۰۸:۳۴


پست: 3268

سپاس: 5491

جنسیت:

تماس:

Re: سوال المپیاد ریاضی

پست توسط rohamavation »

همانطور که معادله معادل شما نشان می دهد، حداقل یکی از اعداد زوج است، و حداقل یکی از آنها باید بر 5 بخش پذیر باشد. اکنون 0.5+0.2=0.7، 0.4 باقی می ماند تا با دو کسر مصری (EF) جبران شود. با شروع با کوچکترین مخرج، یعنی 3، ما در حال حاضر خوش شانس هستیم: 13+115=0.4. آیا می توانیم بهتر از این کار کنیم؟ هدف ما برای شکست 2+3+5+15=25 است. بنابراین ما به دنبال چهار عدد صحیح مثبت هستیم، شامل حداقل یک عدد زوج و یک مضرب از 5، که به 24 یا کمتر اضافه می شود، که مجموع متقابل آنها 1.1 است.
واضح است که 1 را می توان حذف کرد، زیرا این امر باعث می شود که ما مجبور باشیم سه EF را با 0.1 پیدا کنیم و مجموع مخرج از 30 تجاوز کند. بعد 2 می آید (مانند قبل). که یک مجموع EF سه ترم 0.6 باقی می گذارد. تشخیص $\frac13+\frac16+\frac1{10}=0.6$ آسان است. هدف جمع عدد صحیح ما اکنون 20 یا کمتر است. توجه داشته باشید که اگر هر عددی دارای ضریب توان اول باشد که 10 را تقسیم نمی کند، عدد دیگری نیز آن ضریب را دارد. بنابراین ما می توانیم 7،8،9، هر یک از 11،12،13،14،15،16،17،18، و 19 و اعداد بزرگتر را فراموش کنیم، زیرا گنجاندن هر یک مجموع ما را به 20 می رساند. برای بازی باقی مانده است 2،3،4،5،6 و 10. واضح است که
$\frac12+\frac13+\frac16+\frac1{10}=1.1$
را نمی توان بهتر کرد.
تصویر

ارسال پست